subject
Physics, 16.11.2020 16:50 hey743358

Sabiendo que el indice de refracción dela gua es 1,33 calcula el ángulo de refracción resultante para un rayo de luz que incide sobre una piscina con un ángulo de incidencia de 50º

ansver
Answers: 1

Another question on Physics

question
Physics, 22.06.2019 14:40
The experiment done in lab is repeated, using a ball that has unknown mass m. you plot your data in the form of f 2 versus m/l, with f in rev/s, m in kg, and l in m. your data falls close to a straight line that has slope 3.19 m/(kg · s2). use g = 9.80 m/s2 and calculate the mass m of the ball.
Answers: 1
question
Physics, 22.06.2019 15:00
Give an example in which the electrical energy changes to light energy
Answers: 1
question
Physics, 22.06.2019 17:40
Along wire carrying a 4.2 a current perpendicular to the xy-plane intersects the x-axis at x=−1.7cm. a second, parallel wire carrying a 3.0 a current intersects the x-axis at x=+1.7cm. part a at what point on the x-axis is the magnetic field zero if the two currents are in the same direction? express your answer to two significant figures and include the appropriate units.
Answers: 2
question
Physics, 22.06.2019 20:20
Consider a file currently consisting of 200 blocks. assume that the file control block (and the index block in the case of indexed allocation) is already in memory. calculate how many disk i/o operations are required for contiguous, linked, and indexed (single-level) allocation strategies, for each of the conditions listed below. in the contiguous-allocation case, assume that there is not room to grow at the beginning but there is room to grow at the end. also assume that the block information to be added is stored in memory.
Answers: 3
You know the right answer?
Sabiendo que el indice de refracción dela gua es 1,33 calcula el ángulo de refracción resultante par...
Questions
question
Mathematics, 19.10.2020 04:01
question
Mathematics, 19.10.2020 04:01
Questions on the website: 13722360